Sunteți pe pagina 1din 80

Case I

This case is a common and also interesting presentation to pediatricians


(sorry for not writing full details, but only writing what makes the clue
for diagnosis)

A mother came to you with an anxious face and trembling words telling
that her male baby 2 months of age started to vomit blood after each
feeding since one week . She is afraid because her father has died of
hematemesis . Taking full history you reported unremarkable perinatal
history (ante, natal and postnatal ) the baby is a product of full term
normal vag delivery, normal birth weight, no associated illness or
foreign body ingestion or bleeding tendency manifestations whether
skin, urin, stool or from other orificies, no associated trauma, average
developmental milestone, consanguinity is positive, no family quarrels
or abuse, good social standard and the mother is educated , nutrition is
in the form of exclusive breast feeding, Examining the baby you
reported a lovely active playful baby , appears well of average built and
nutritional state with average percentiles on growth curve, vital signs
are normal, no pallor or jaundice or cyanosis, no enlarged lymph nodes
or organomegaly, chest and heart are free , skin and joints are normal
?What simple question will you ask that lady about??
?What is your probable diagnosis according to what is common?

This is a Case of Swallowed Blood Syndrome, due to fissured nipples
This will be confirmed when you ask the mother whether she feels pain
during nursing her baby as she may also overlook the nipple fissures
that open only during infant sucking . Maternal blood is surely different
from fetal blood that contains more Hb F that is resistant to alkali
denaturation. When you advise ointments and antiseptics the condition
resolves safely. I mean this the common form of what is called
pseudohematemesis in exclusive breastfed babies. you have to exclude
other local or systemic causes of bleeding disorders if the case merits.
Other form of pseudohematemesis or swallowed blood syndrome in
grown up children comes from swallowed epistaxis blood when it is
dirested postnasally. This case in fact has an alarming presentation
but the treatment is simple assurance and advice for the mother to
having soothing local remedy and continue breast feeding












Case2

A lady 40 years of age came to you asking your advice on what she
has heard in the media that an elderly mother has a risk of
delivering a mongol baby? You took a deep breath and told her Yes
the probability in general population is ------- and at your age is -----
-----------------Then you advised doing a blood test, when the results
came you told her she is in need of doing other confirmatory test to
get 100% sure result? What is your comment on probabilities, blood
test , the 100% confirmatory test, the name of the syndrome is it
correct to call it Mongolism, Downs Syndrome, Down Syndrome, or
Down's Syndrome.

As for case 2 Down syndrome (this is the right nomenclature) have a
general population incidence of 1/700 (i-e among each 700 pregnancies
this is a probability of one getting DS) this probability however differ
with different ages to be 1/900 for young women <30years to 1/350 at the
age of 35ys and 1/100 in our case of the lady aged 40y. The blood test is
the Triple Test: in which you measure 3substances in maternal serum
these are alpha feto protein, serum esteriol (both become low in preg
with DS)and serum chorionic gonadotrophin or CGH (which becomes
high in preg with DS) these data if coupled with the lady age, preg
gestation, presence or absence of diab give a probability of DS of about
70% and if coupled with measuring inhibin A (quadrible test) and fetal
U/S for nuchael line, short femur will increase the probability. However
to have a probability of 100% you have to do karyotyping of fetal tissues
(ie chromosomal analysis) through amniotic fluid samples got with
amniocentesis or fetal tissue through chorionic villus sampling CVS and
find the trisomy21 ie 3copies of ch21





















Case 3

Six day old baby girl born vaginally after 30 weeks gestation with
persistently enlarged huge solitary gastric bubble, no distal air in the
KUB (see attached figure).
She passed some meconium on the first day of life. After 24 hrs of life
she developed hypochloremia.
The UGIS shows a dilated stomach, a normal antrum with a cap and faint
insinuation of a pyloric canal



As for case 3 I have to greet the truly magnificient DrMagnificient for her
accurate thinking but take care that this case had an early presentation
at 8 days and had chracteristic pyloric obstruction consistent with
pyloric *** and atresia this will be compared to the more common
congenital hypertrophic pyloric stenosis CHPS that usually presents at
least at the age of 5-8 weeks ie after the first month of life























Case 4

A male Child aged 3 years presented to you with just a scratching
sensation of his throat but his mother was worrid about the skin rash
that is spreading over his body even his face. History of present illness
showed no remarkable symptoms with only mild brief fever, no runny
nose or cough and no other symptoms referable to other body systems.
The same is true for his perinatal, developmental ,, nutritional social and
vaccination and past history all are within the normal range.
Examination showed a playful interactive ***** boy normal intelligence
speech gait and posture no pallor, jaundice or cyanosis average built
and nutrition vital signs showed temp 37.3oC, pulse regular 80/min not
faint or collapsing RR 19/min and blood pressure 90/60 no dysmorphic
features, throat slightly congested and ear drums are normal, chest and
heart examination free abdominal examination no abnormality could be
detected However skin examination showed an erythematous rash
macular lacy appearance allover the body and exterimities, the cheeks
also appeared reddish with no itching marks or oedema joints were also
free. (see attached figure )
What is the most probable diagnosis??? and give your differential
diagnosis of such skin rash




Erythema infectiosum, or "fifth disease", is a common infection of
childhood caused by a virus. parvovirus B19, a virus that lives only in
humans. It seems to be transmitted mainly by body fluids, including
droplets produced when you cough or sneeze but also including blood.

The term "fifth disease" does not refer to someone named "Fifth", but
rather to the fifth of six classic exanthems, or rash-associated diseases,
of childhood.
The numbering is of historic interest only:
the other examthems, in order are
first: measles;
second: scarlet fever;
third: rubella;
fourth: "Dukes' disease", which was never clearly distinguished from
other rash-producing diseases and is now thought to have been either
measles, rubella, scarlet fever,
a Staphylococcal infection (?toxic shock syndrome), or one of several
unspecified enteroviral infections;
and sixth: roseola infantum .

Symptoms usually appear within 4 days to 2 weeks after exposure,
The virus seems to be less con***ious once the rash appears (so
children with the rash can go to school or day-care without exposing
others).

The most common symptoms of fifth disease are
mild cold symptoms and malaise,
fever in up to 1/3 of patients,
and a rash.
On the body this rash is usually light red and lacy in appearance -- the
typical rash does not look like measles,
The best-known rash of fifth disease is on the face, which becomes
intensely red, especially on the cheeks, with a pale ring around the
mouth ("circumoral pallor").
The face looks like the patient's cheeks have been slapped
(
In addition, joint pain can occur: it's relatively rare in kids but more
common in adults.
Many people (adults and children) are infected with parvovirus B19 and
show no symptoms whatever.

rare complication is anemia caused by hemolysis (breakdown of red
blood cells).
People with abnormalities in their hemoglobin, such as those with
sickle-cell disease, may develop an "aplastic crisis".

It is also possible for parvovirus B19 infection during pregnancy to
cause "hydrops" (

On the other hand,
Hand, foot, and mouth disease -,=>s\! _=>\!
is usually characterized by tiny blisters on
the inside of the mouth and the palms of the hands, fingers, and soles of
the feet. It is commonly caused by coxsackievirus A16 (an enterovirus),
and less often by other types of viruse














Case 5 : OSCI case


Now we come to today's case : please look carefully to the
provided pictures and brief di******ion and give your most
probable diagnosis

Mucocutaneous lesions in a girl of 10 years presented with low
grade fever, anorexia, malaise and weakness, since 2 months




Firstly : a female (in lupus female to male ratio is 8:1)
secondly: age of 10 years (peak age incidence around 10 -15 years
thirdly: chronic systemic manifestations of fever, malaise and anorexia (common
in SLE)
fourthly the characteristic mucocutaneous rash
erthymatous rash butterfly distribution over malar eminence (sometimes with
photosensitivity), muculopapular rash, mucous mem ulcers (usually painless)

Yes you have to search for other system affection especially hematologic system
(hemolytic anemia, leucopenia and thrombocytopenia), joints (arthritis or
arthralgia of small joints knee and wrist), respiratory system (pleurisy and
effusion), cardiovascular system (pericarditis, myocarditis endocarditis or
Libman-Sachs disease and hypertesion), neurologic system (ataxia,n palsy,
regression, chorea), renal involvement (nephritis, hematuria renal hypertension),

These if coupled with autoantibodies that will make the diagnosis confirmed eg
ANA that is positive in most of the cases but anti double stranded DNA or anti
dsDNA and anti Smith antibodies are more specific

Diagnosis: ACR (American College of Rheumatology meeting) requires 4 of 11
criteria
Malar Rash
Discoid rash
Photosensitivity
Oral Ulcers
Polyarthritis involving more than 2 joints
Pleuritis or Pericarditis
Antinuclear Antibody titer positive (1:40 or higher)
Titer over 1:320 is very suggestive
Renal disease
Neurologic disorder (e.g. Seizures, Psychosis)
Anemia, Neutropenia or Thrombocytopenia
Anti-dsDNA, Anti-Sm positive, Syphilis False Positive






































Case 6

A lady came to you taking your advice should she proceed to cicumcise
her newly born male baby aged 7 days??. She is worrid because her
family are known to be a hemophilia family. She had 2 affected brothers
and one maternal uncle who died of bleeding tendency and diagnosed to
be Hemophilia A disease. Perinatal history was Ok The baby didn't
experience bleeding from the umblicul stump or cephalhematoma or
from orifices and is exclusively breastfed, Examination of the baby
showed that all systems were free except for the observation of a
greenish blue area in his lower back that his mother pointed to it with
high suspecion of being abnormal. You soon ordered blood tests for
both the baby and his mother. When the results came you advised the
mother not to worry and to go on immediately for circumcision.
What are these tests you have ordered?
What are thier relevance to the baby and also to the mother?
Why did you advise circumcision inspite of this skin affection?

As for case 6 we observe that the mother is coming from a family
known to be a hemophilia A family. This woman had 2 affected
brothers and one maternal uncle. Her mother will be an obligate
carrier of the disease. giving the probability of 50% to her
daughters to be carriers of the disease. So this woman has a
probility of 50% to be a carrier (taking the assumption that her
father is not hemophiliac).
Now how can you know that this woman is a carrier for hemophilia
A disease??
By doing blood clotting factor VIII assey and if feasible DNA test
In carriers factor VIII will range from 30-70% which usually gives
no symptomts. In comparison to the case manifesting bleeding
diatheses together with other manifestations of the disease at a
level below1% although manifestation vary within the range of 0 to
5%
Ordered blood test for the baby and his mother therefore will
include clotting profile (PT, PTT, CT and factor VIII assey)
The results of these tests were within normal limits so the
conclusion was that this mother is not a carrier and her baby was
not affected. and so she has to go on for circumcision.
But what about this skin bluish green spot????
I will be back with you after a while

This skin spot coloured greenish blue, or grayish blue or blackish blue sometimes
mistaken for bruises are normal spots called Mongolian Spots usually found on
the buttocks and lower back and sometimes on limbs especially in dark skinned
infants Found in normal newborns and usually disappear by childhood.


By the way How many types of Hemophilia we know and what types of
inheritance pattern they follow?

1) The most common form of hemophilia is vonWillibrand disease which is
autosomal dominant and which has abnormal bleeding time due to defective
platelet aggregation due to deficiency of vonWillibrand factor which is a part of
factor VIII (factor VIII antigen)

2) Hemophilia A due to deficiency of factor VIII inherited in an X-linked
recessive pattern

3) Hemophilia B or Christmas disease due to deficiency of Christmas factor or
factor IX and also is inherited as X-linked recessive pattern

4) Hemophilia C due to deficiency of factor XI and is inherited as autosomal
recessive pattern

5) Parahemophilia due to deficiency of factor V and is inherited as autosomal
recessive pattern


















Case 7 OSCE

4month female child
with a right upper quadrant abdominal mass and jaundice.
Ultrasound (US) and nuclear scintigram (HIDA) are included
.


Choledochal cyst is a cystic dilatation of the common bile duct from
congenital defect of the duct wall or a mucosal valve.
Choledochal cysts, occur in girls 4 times more frequently than in boys.
Incidence is 1 in 2 million live births.
It is more frequently observed in Asians. 50 percent are diagnosed in
infancy.

They are classified type I to type V

Type I The commonest one: Diffuse enlargement of CBD

II & III Affects extra hepatic bile ducts
IV Multiple Intra and extra Hepatic Cysts
V Also called Caroli's disease is a ductal plate malformation with
intrahepatic dilated iliary tree.













Case 8

JJ a 3-month old infant presented with vomiting since the first
week of life. The vomiting occurs daily, is usually small in
amount but occasionally is projectile. Physical examination
reveals a happy infant who is thriving well. The breathing is
noisy and an occasional cough is heard.
???????What is your suggested approach based on suspected
diagnosis


The history is consistent with a diagnosis of gastrooesophageal reflux
or GER which is almost physiological in the first 3 months of life
Posture and thickening the fluids is the treatment of choice
A chest X-ray may indicate aspiration pneumonia
particularly if cough and wheeze are associated with feeding

On the other hand, infants with pyloric stenosis fail to thrive, lose weight
and have projectile vomiting
They almost never reach the age of 3 months without surgical correction
GER diagnosis is made on the clinical history
Occasionally is confirmed by barium studies which have a value of
exclusion of other causes of vomiting as well.

























Case 9

NN is a 12 ys old boy, known case of sickle thalassemia. He
came to the ED with a 24 hours history of pain in his right
leg below the knee associated with difficulty walking. He
was slightly jauniced and his spleen was pabable 3 cm below
the left costal margin. His temp was 38oC.
What is your comment on the following:
need for hospital addmission,
radiology on the affected limb,
need for analgesia,
fluids,
other important investigations.
This case of sickle thalassemia means that there is double
heterozygosity for sickle cell diseas and thalassemia
Also means that his parents are carriers for the disease one is a carrier
for sickle cell disease and the other for beta thalassemia
Also means that this subject has both Hb S and increased level of Hb A2
This subject has microcytic hypochromic anemia of beta thalassemia
with target cells (note that SCA is characterized by being normochromic
normocytic)
This case is usually complicated by thrombotic or infarctive crisis,
hemolytic crisis, aplastic crisis or sequesteration crisis
In the present case bone infarction (thrombotic crisis) is the most likely
diagnosis In that case the following is needed
Admission to the hospital, adequate hydration, probable use of
analgesia
blood culture for excluding infection (also a common complication) ie
osteomyelitis especially with gram negative organisms that warrants the
use of antibiotics
As for radiology, no need to hurry as it is usually negative in the acute
stage









Case 10

An 8-month-old female is brought into the emergency
department by her mother. The infant was in her usual
state of good health until three days ago, when her
mother noted that she felt warm. She was less active,
but eating well and had no vomiting, diarrhea, or other
complaints. The following day (2 days ago), she
refused to eat. She continued to have fevers and
remained less active than normal. That evening she
was taken to the ED and was found to have a
temperature of 39.5 degrees C. She was described by
her mother as being less active, not eating well, and
having a decreased urine output. CBC showed a white
count of 12.1 (32 segs, 2 bands, 53 lymphs, 9 monos),
hemoglobin 11.1, and hematocrit 32.8. A blood culture
was obtained. A chest radiograph was felt to be
normal. The infant was given antipyretics and
discharged home.
The next day (yesterday), she continued to have
fevers. Her temperature was 38.5 degrees C. She was
less active and was not eating or drinking. She had not
voided according to her mother. There was no
vomiting, diarrhea, or respiratory symptoms. Her
mother also noted on this morning that she had some
skin lesions which resembled insect bites. They were
red and raised. She was taken to see her primary care
physician, who evaluated her and discharged her.
She is brought to the ED today because of
increasing lethargy, very poor oral intake, no urine
output for 24 hours and continued fever.
Past Medical History: She was born via normal
spontaneous vaginal delivery. Prenatal and postnatal
courses were uncomplicated. She is usually on formula
but has taken some solid foods. Her immunizations are
up to date. She had one prior hospitalization at 4
weeks of age for fever during which time a sepsis
workup was negative. Her family history is
unremarkable.
Exam: VS T 39 degrees C, P 160, R 35, BP 95/46.
She is very lethargic. She is not cooing, smiling, or
vocalizing. She has a weak cry and is tachypneic. She
does not respond to painful stimuli. She assumes a
position of comfort, in which her lower and upper
extremities are flexed and her head is turned to the
right. Attempts to place her head midline result in a
weak cry. There is slight nuchal rigidity. She is pale
with mottling of her lower extremities. There are
multiple skin lesions across her face and a few on her
arm. They are irregularly crusted (questionably
pustular) centrally, surrounded by some erythema. She
appears to have a facial droop on the right. Her
anterior fontanelle is small but sunken. Her mucus
membranes are sticky and she does not produce any
tears when she cries. She does open her eyes, but will
not follow objects and has a dysconjugate gaze. Breath
sounds are bilaterally coarse. She has some upper
airway rhonchi. She has no wheezes, crackles, or
rales. Heart regular without murmurs. Abdomen flat
and soft. Radial pulses are 2-3+ bilaterally and equal.
Her hands and feet are cool.

An IV is started and IV fluid is infused. She is also
given an IV dose of ceftriaxone. She begins to have
extensor posturing with arching of her back. She keeps
her head to the right. She also develops tonic-clonic
movements of her left upper extremity, and is therefore
given a IV lorazepam followed by phenytoin.

What work-up would you recommend?

Because of the infant's fever, nuchal rigidity,
lethargy, and focal neurologic abnormalities, a
diagnosis of meningitis and/or encephalitis is
suspected. A lumbar puncture is performed which
shows fluid that is questionably hazy (minimally). She
is given IV acyclovir and vancomycin (ceftriaxone was
given earlier). The laboratory analysis of the CSF
shows 105 RBCs and 40 WBCs per cubic mm with 10%
segs, 80% lymphs, and 10% monos, glucose 75,
protein 59. The Gram stain shows no organisms.

??????What additional workup will you advise for diagnosis and
management


This case is most probably consistent with : Herpes
meningeoencephalitis

These clinical and laboratory findings have the following DD
:
Aseptic meningitis
Bacterial meninigitis
Tuberculous meninigitis
Viral encephalitis
Brain abscess
Cerebrovascular accident
Shaken infant syndrome

The CSF analysis is consistent with aseptic
meningitis, but since this is usually a benign clinical
entity, this child who is very sick, is not likely to have
just aseptic meningitis.
Bacterial meningitis is not likely
(practically not possible) given this CSF analysis.

Tuberculous meningitis is possible, but the clinical
course is too acute for this.

Viral encephalitis is the
most likely since the CSF analysis suggests a viral
picture and the patient's poor neurological status is
consistent with an acute encephalitis.

Additionally, the
skin lesions on the patient's face may be herpes
simplex lesions.

Brain abscess is another possibility
which needs to be ruled out by imaging.
Cerebrovascular accident cannot be ruled out, but it is
less likely.

Shaken infant syndrome is usually
associated with CSF which is grossly bloody.

Brain
imaging studies will help to obtain a more definitive
diagnosis.

A CT and a MRI are performed.
The CT scan without contrast demonstrates
some hypodensity in the thalami and
temporal lobes, indicative of bilateral cortical and
subcortical edema.

After the administration of IV
contrast (lower two images), there is meningeal
enhancement at the margins of the abnormal
parenchyma, as well as mild central enhancement.
This
is consistent with a breach in the blood-brain barrier
following vasculitis.

There is no evidence of herniation.
The areas of increased signal intensity in the MRI
scan indicate the presence of abnormal water,
consistent with the CT scan findings of
cortico-medullary edema in the bilateral
temporoparietal lobes.

There is also meningeal
enhancement over the affected areas of the brain.
The above findings are compatible with bilateral
temporoparietal lobe meningoencephalitis.


The CSF sample is positive for herpes simplex virus
(HSV) DNA by polymerase chain reaction (PCR)
although the viral culture of the CSF is negative.

Her EEG is abnormal. It is diffusely slow for her
age.
These results are compatible with a diffuse
encephalopathic process.

In addition, there are some
sharp waves emanating from the right frontotemporal
and right frontal central regions, consistent with seizure
foci in the affected areas shown on her brain CT and
MRI.
CSF analysis in cases of bacterial meningitis is characterized with
high cellular ******* ~1000 celll/mm3mostly pus cells ie neutrophils
with a low suger ******* (ie ~ 40% of blood suger )
whereas normal CSF suger is ~60% of blood suger with increased
protein ******* (nomally 50mg/ml)
in aseptic and viral menigitis etiology cell count is moderately elevated
ie ~100 per mm3 mostly lymphodcytic and with normal suger *******
except in some viral infection like mumps and also increased protein
The same is also recorded for TB meningitis with increased monocytic
cells and decreased chloride *******
The gold standard tests FOR DIAGNOSIS is culture for recovering
the offending organism
PCR is now replacing cultures for diagnosis
The most common form of viral meningitis is due to Enteroviruses
(ECHO, Coxaskie and polio)
The most common sporadic form of viral meningitis is due to Herpes
Simplex virus 1 (IN NEONATE BOTH 1 AND 2 TYPES) hence the
emprical use of acyclovir once viral meningeoencephalitis is suspected
like our case
The most common and severe form of epidemic viral meningitis is due
to ARBO viruses carried by vectors as mosquitoes and ticks












Case 11

A one year old female child presented with history of vomiting and
diarrhea for 3 days and decreased urine output of 1 day duration. On
examination, the child was sick looking. Pallor was marked. Anterior
fontanelle was depressed and the skin turgor was reduced. The pulse
could not be felt and the respiration was deep and sighing with a rate of
60 per minute.
An intravenous access was obtained immediately and 2 boluses of
normal saline were given.
But the pulse couldn't be felt.
An ECG was obtained
Immediately,
drugs A and B were administered and the pulse could be felt.
A repeat ECG was taken after 10 minutes
After a period of 15 minutes, another ECG was taken
which showed a few changes .
A drug C was instituted and the ECG changes disappeared gradually
after a period of approximately 2 hours .


What was the most probable diagnosis?
What were the drugs A, B and C?
What were the ECG changes seen in this condition?








a case of acute severe hyperkalemia

The normal potassium level is 3.5-5.0 mEq/L, and total body potassium
stores are approximately 50 mEq/kg. Hyperkalemia is defined as a
potassium level greater than 5.5 mEq/L.
Ranges are as follows:
5.5 - 6.5 mEq/L - Mild condition
6.5 - 7.5 mEq/L - Moderate condition
7.5 mEq/L and greater - Severe condition

However, it should be remembered that the clinical manifestations of
hyperkalemia depend not only on the absolute value but also the rate of
rise in serum potassium level.

Hyperkalemia may results from the following:
Decreased or impaired potassium excretion -- Decreased glomerular
filtration rate (acute or end-stage chronic renal failure), decreased
mineral corticoid activity (Addison disease), defect in tubular secretion
(renal tubular acidosis IV), drugs (NSAIDs, cyclosporine, potassium-
sparing diuretics).

Additions of potassium into extracellular space - Exogenous
potassium supplementation (potassium replacement orally or
parenterally, particularly when combined with drugs that impair
potassium excretion), rhabdomyolysis, and hemolysis (massive blood
transfusions, exchange transfusion, burns, tumor lysis syndrome).

Transmembrane shifts (ie, shifting potassium from the intracellular to
extracellular space) - acidosis, genetic (Hyperkalemic familial periodic
paralysis), insulin deficiency, and medication effects (eg, acute digitalis
toxicity, beta-blockers, succinylcholine).
Factitious or pseudohyperkalemia - As observed with improper blood
collection (eg, ischemic blood draw from venipuncture technique),
laboratory error, leukocytosis, and thrombocytosis.

ECG changes in hyperkalemia [Table 1]
Serum K+ ECG Changes
5.5 - 6.5 Peaking of T waves
6.5 - 7.5 QRS widening
7.5 - 8.5 Decreased amplitude of P wave and Increasing P-R interval
> 8.5 P wave disappears, "Sine wave" appears, Ventricular fibrillation,
asystole

Treatment


The urgency of treatment depends on the severity of hyperkalemia. Mild
hyperkalemia can be treated with reduction in potassium intake,
discontinuance of drugs such as potassium-sparing diuretics, beta-
blockers, NSAIDs, or ACE inhibitors, which tend to cause hyperkalemia
and addition of loop diuretics. Moderate to severe hyperkalemia requires
more aggressive therapy. However, in acute or chronic renal failure,
especially in the presence of hypercatabolism or tissue injury, treatment
should be initiated when the plasma potassium level exceeds 5 mEq/L.

Treatment of acute hyperkalemia

In emergencies such as cardiac toxicity or moderate to severe
hyperkalemia, the following measures should be performed immediately
in rapid sequence without waiting to repeat plasma potassium values.
1.
Intravenous administration of I - 2 ml/Kg of 10 percent calcium gluconate
over 5 minutes. Calcium should be given slowly as rapid infusion can
cause cardiac arrest in systole. Caution should be used when giving
calcium to patients taking digitalis because of the risk of precipitating
hypokalemia-related arrhythmias.

2. Intravenous administration of sodium bicarbonate at 1 - 2 ml/Kg over
5 - 10 minutes. Administration of sodium bicarbonate has its associated
problems of hypernatremia and hyperosmolarity.

3. Intravenous administration of 0.1 U/Kg of regular insulin followed
immediately by rapid infusion of 0.5 - 1.0 g/Kg of glucose. The main risk
of insulin - glucose infusion is hyperglycemia.

4. Inhalation or intravenous administration of i:beta agonist. This has
been shown to be efficacious in treating hyperkalemia.














Case 12

An 18-month-old girl with high fever 39.5oC malaise and
anorexia
urine analysis using a bag urine specimen showed 50000
cliforms per ml, white cells 100 per cmm
red cells 10 per cmm and no epithelial cells

What action would like to undertake regarding
antibiotic treatment
another urine bag sample
suprapubic aspiration sample
renal ultrasonography
??????? renal function


The results of the bag urine culture are not diagnostic of urinary tract
infection.
Antibiotics
should not be commenced
until an appropriate specimen of urine has been collected;
at this age the preferred method is by suprapubic aspiration,
although a midstream specimen (clean catch) also may be satisfactory.
Bag urine specimens are often contaminated;
a growth of 50 000 organisms/ml
cannot be taken as evidence of infection.
Even in proven cases of urinary infection,
renal ultrasound generally is deferred until the child is well[/mark].


















13 Case

A 14-year-old boy
presented with history of tiredness, irregular fever and bleeding
tendencies of one-week duration.
There was no history of any illness or any drug intake in the past.
He never had any jaundice or exposure to chemicals.
Family history was also non-contributory.
Physical examination showed moderate pallor, purpuric lesions in the
oral cavity and over the trunk.
There were no congenital anomalies.
Hemogram showed pancytopenia. Hb was 6g/dl, WBC count
1000/Cumm with
neutrophils -10 percent, lymphocytes 90 percent.
Platelet count was 38,000/cumm.
Markers for hepatitis B and C were negative.
Sucrose lysis test was negative.
Bone marrow aspirate was very hypocellular,
biopsy showed features consistent with severe aplastic anemia
Comment on diagnosis and management of this case

Diagnosis : Aplastic anemia
According to the International Aplastic Anemia Study Group Criteria for
Severe aplastic anemia is marked pancytopenia with at least two of the
following: granulocytes less than 500/microliter, platelets less than
20,000/microliter, anemia with absolute reticulocyte count less than
40,000/uL, plus a marrow biopsy showing less than 25 percent of normal
cellularity or a marrow showing less than 50 percent cellularity in which
fewer than 30 percent of the cells are hematopoietic. Very severe
aplastic anemia patients have neutrophil counts of less than 200 per
cubic millimeter.
common causes of Aplastic anemia
a. idiopathic - this is the commonest cause (>50%)
b. drugs - chloramphenicol, benzene
c. viral infections - hepatitis viruses, EB virus
d. congenital - Fanconi's anemia
e. Pregnancy
Differential diagnosis
a. Aplastic anemia
b. Myelodysplastic syndrome MDS
c. Leukemias esp. AML-M3 and hairy cell leukemia
d. Megaloblastic anemia
e. Paroxysmal nocturnal hemoglobinuria PNH
Bone marrow aspiration and biopsy will differentiate Aplastic anemia
from other conditions. In all other conditions except aplastic anemia,
bone marrow will be hypercellular. Rarely in hypocellular variety of MDS
also bone marrow may be hypoplastic. In such situations, dysplastic
changes in the myeloid and erythroid cells and cytogenetic studies will
help to differentiate between the two.
Investigatigations
The first thing to do is a complete hemogram and peripheral smear.
The usual findings are - normocytic or macrocytic red cells with
leucopenia and thrombocytopenia. Dysplatic features are absent. If
present that points to a diagnosis of MDS. Also screening for hepatitis B
and C and paroxysmal nocturnal hemoglobinuria (Ham's test, Sucrose
lysis test and if available flow cytometry for CD 51) is required. Bone
marrow aspiration and biopsy and karyotype is need to exclude
Fanconi's anemia. Excessive chromosomal breakage (mitomycin
induced) is suggestive of Fanconi's anemia. Once aplastic anemia is
confirmed they need HLA typing to see whether a suitable match for
bone marrow donation is available.
The treatment of choice in this boy is allogenic bone marrow
transplantation (BMT). Success of BMT in this age group range from 40
to 90 percent in the best centers. However, since this boy is not having a
complete match (ie. 6/6 match) we can consider only an unrelated
transplant in this patient. However, there are problems with unrelated
BMT, like increased procedure related mortality and graft failure. So an
alternative approach most suitable for this patient is to go for
you can go for unrelated , If it fails . apy first immunosuppressive ther
. BMT
Complications of BMT
on the average is , after HLA identical sibling transplant : failure - graft . a
10 percent. Older patients are at increased risk of graft failure. Exposure
to large number of transfusions and long duration of the disease
increases the chances of graft failure.
It may be . This is mediated by the donor T cells : Graft Vs host disease . b
of 2 types- acute or chronic. Acute GVHD is characterised by skin
lesions, gastrointestinal changes and liver dysfunction. Features of
chronic GVHD resemble an autoimmune disorder like scleroderma.
Immunosuppressive treatment
The most commonly used agent for immunosuppression is
antithymocyte globulin (ATG). Others are antitlymocyte globulin (ALG),
cyclosporine and steroids. About 50 percent of patients will respond to
ATG alone, and about half of the remaining patients have some
response to cyclosporine. When the two agents are combined, as many
as 60-78 percent of patients respond hematologically. ATG is more
effective in patients with moderate aplastic anemia, compared with
patients with very severe aplastic anemia. But the problems are relapse
and evolution to paroxysmal nocturnal hemoglobinuria (PNH) that has
been seen in 10 percent of these patients. ATG/ALG and transplantation
are costly. Cyclosporine alone is also found to be effective especially in
the subsets of patients with moderate aplastic anemia




Case 14

This is a 5-month old male with fever, irritability, and
vomiting. His temperature at home was 38.0 rectally.
He vomited five times since onset 8 hours ago. He is
feeding soy formula. Past medical history is
unremarkable.
Exam: VS T38.0R, P150, R40 (crying), BP
unobtainable, wt 50%ile. Fussy, though consolable at
times. He doesn't focus or interact well and appears
somewhat "toxic" to the examiner. Anterior fontanelle is
somewhat full, but he is crying so it is difficult to truly
assess. Eyes moist. Pupils reactive. TM's are shiny
and slightly red. Oral mucosa moist. Neck is hard to
assess due to crying. Heart regular without murmur.
Lungs clear. Abdomen soft on inspiration. No
detectable tenderness evident. No hernias. Testes not
swollen. CVA tenderness is not apparent. Color
slightly pale. Capillary refill time 2-3 seconds. Muscle
tone good.
A lumbar puncture is done to rule out meningitis.
The CSF is homogeneously bloody (blood mixed with
CSF). The blood does not clear. All three tubes
appear to be equally bloody. A CBC, blood culture, and
catheterized urine sample are sent to the lab. An IV is
started and the child is given 50mg/kg of cefotaxime IV.
A CT scan is done to rule out subarachnoid
hemorrhage.
What is your probable diagnosis and differential
diagnosis?????????????

Shaken Baby Syndrome

A full fontanelle is not always indicative of meningitis. Intracerebral
hemorrhage, cerebral edema, and acute hydrocephalus can all mimic the
same clinical features.

Although the falx may enhance with IV contrast, an increased density of
the posterior falx before IV contrast is administered should raise the
suspicion of a posterior interhemispheric subdural hematoma. This
injury is highly indicative of a shaken baby. Other findings such as
retinal hemorrhages and a suspicious history add to the strength of this
etiology.


An experienced physician who has done many lumbar punctures in
infants usually knows when to expect bloody CSF due to the difficulty of
the procedure. If bloody CSF is unexpectedly encountered, and it does
not clear, one should be highly suspicious of intracerebral hemorrhage
with blood entering the subarachnoid space.

Although blood should not appear in the CSF if the hemorrhage is
purely subdural, this injury is not purely subdural in nature. Blood also
enters the subarachnoid space. Axonal shearing and generalized
cerebral cellular injury take place as well.


Trauma specialists have often taught that intracranial hemorrhage alone
cannot account for all the blood loss in a patient in hypovolemic shock.
In other words, if you have diagnosed an intracerebral hemorrhage in a
trauma patient in hypovolemic shock, you must look elsewhere for
additional hemorrhaging sites, such as in the abdomen. Infants appear
to violate this rule since many shaken babies present to the emergency
department in shock. Although one must always be suspicious of other
hemorrhaging sites from injuries such as from fractures and internal
injuries, subsequent work-ups on these patients may fail to identify
significant hemorrhaging sites other than in the brain.

CT scans may fail to show a posterior inter-hemispheric subdural
hematoma if it is small. MRI scanning has been shown to be more
sensitive at identifying these hemorrhages and other brain injuries















Case 15

This 3 year 6 months old male child presented with multiple, tiny,
pinkish red nodules on the face for the last nine months. These acne like
skin lesions had very slowly increased in number. : Photograph of the
child showing facial angiofibromas and forehead plaque

Treatment with steroids and calamine lotion did not help.

The child was apparently normal with weight 15kg and height 98cm
(both between 25th and 50th percentiles). He had normal growth,
development and intelligence, and was born by LSCS. There was no
history of seizures.
Detailed examination revealed few hypopigmented patches on the
forehead, face and back. The patch on the back was leaf-shaped. He also
had skin colored thickening of the skin on the lumbo-dorsal back, which
had orange-peel consistency).
Fig. 2:Photograph of the child showing ash leaf lesion at back and
Shagreen patch over lumbosacral area.

Except for these skin lesions, general physical examination and
systemic examination did not reveal any clinical finding.

The acne like lesions, the hypopigmented patches and the lumbosacral
lesions were suspected as adenoma sebaceum, ash leaf and shagreen
patch respectively.

Investigations

Cranial CT scan was advised. Since the patient was asymptomatic, the
parents were reluctant for investigations. However, in 4 months, the
patient had a fall, with head injury and fracture lower end of left radius. A
cranial CT scan done on this occasion showed multiple tubers, the
largest lesion being in the left temporal lobe (12mm x 8mm) and ill
defined lesions in right temporal and left frontal lobe, and subependymal
lesions.

Cranial MRI done on 20.11.00 showed evidence of diagnostic calcified
subependymal nodules and multiple focal areas of hyperintense signals
in both cerebral hemispheres . There was hypointensity in white matter
suggestive of incomplete myelination. Some of the subependymal
tubers were projecting into the ventricles producing a characteristic
candle-dripping appearance ().

Ultra sound of the abdomen was done on 1.12.00 and showed the liver to
be mildly enlarged with minimal fluid in the pelvis. Kidneys, adrenals
and pancreas were normal.
X - Ray Chest was normal. There was no evidence of cardiomegaly.
Echocardiography ruled out any associated cardiac
rhabdomyosarcomas.
EEG was abnormal and showed left frontotemporal interictal spikes.
Fundal examination showed a normal retina.
What is the likely Diagnosis?
see attached figures to help you in resolving the case







Tuberous Sclerosis (TS) or Bourneville disease

is due to a gene defect on

chromosome 9q (TSC-1)
or 16p13(TSC-2).
T
he mutated genes code for tuberins,
proteins that modulate the GTPase activity of other cellular
proteins.

It is inherited as an autosomal dominant trait
with an estimated frequency of 1 in 30,000.

It is a neurocutaneous syndrome with a wide clinical spectrum,
but the typical triad

is of seizures, mental retardation and skin lesions.
It is a disorder of cellular differentiation and proliferation
and abnormal neuronal migration, and can affect almost any
organ
except the skeletal muscle.

Brain: The characteristic brain lesions
consist of tubers in the subependymal region,
where they undergo calcification and project into
the ventricles producing a candle - dripping appearance.
Tubers in the region of the Foramen of Munro may cause
obstruction of CSF and hydrocephalus.

Epilepsy is a common manifestation
seen in 80 - 90 percent of patients and
may present as infantile spasms with a hypsarrhythmic EEG
tracing.
Older children may develop complex partial seizures or other
focal seizures.
Mental Retardation occurs in 60 - 70 percent of patients,
and the severity varies.
It is often associated with seizures.

Astrocytomas of the subependymal
giant cell variety are seen in 10 percent of patients,
usually in the first or second decades.
Retinal lesions are of 2 types:

a. Refractile. Yellowish, multinodular cystic lesions
called Mulberry tumors from the nerve head.

b. Round and flat grey color lesions (Phakoma)
in the region of the disc.
Occasional patients have a pigmentary defect of the iris.
Progressive visual loss does not usually occur.

Skin Lesions:
a. Adenoma Sebaceum are angiofibromas of the face.
They are smooth, red,
telangiectatic 1-10mm papules seen in 80 percent of patients in
childhood.

b. Shagreen Patches are large skin colored
irregularly thickened plaques with an orange peel or
cobblestone texture usually in the lumbosacral area.

c. Ash Leaf lesions are hypopigmented macules
on the trunk, face and limbs usually from birth, seen in 90
percent of patients.
An ash leaf spot on the scalp results in Poliosis,
a circumscribed patch of grey white hair.

d. Periungual Fibromas are firm flesh colored
growths on the nail folds seen at puberty.

e. Caf au lait spots may occur.
Heart: Cardiac Rhabdomyomas are present in 50
percent of infants with TS and are usually asymptomatic,
unless they are intraluminal and cause obstruction or
arrhythmia.
Kidney: Renal Angiomyolipomas are seen in 50 - 80 percent
patients;
hamartoma or polycystic disease may also be seen.
Clinically, these lesions present as hematuria,
renal failure or as nephrotic syndrome.

Lungs: Pulmonary Angiomyolipomas are seen in only 1 percent
of patients, and are five times more common in females,
producing cystic or fibrous changes leading to spontaneous
pneumothorax.

Other Organs: Angiomyomas or hamartomas of any organ,
especially adrenal, liver or pancreas may be seen.
Hamartomatous rectal polyps are useful for histologic
diagnosis.


Case 16

A 5-year old girl presented with short stature,
She has always been short but this has become more apparant
over the last year
Her height is less than 2nd centile
And her weight is 2nd centile
She is asymptomatic
She is not dysmorphic
??????What would the most useful investigations be

The first diagnosis you have to exclude in this girl presenting
mainly with short stature is : Turner syndrome
Chromosomal analysis is the primary investigation
in Turner syndrome
Chromosomal pattern is 45,XO or monosomy X
ie 45 chromosomes with XO sex chromosomes
Another forms are mosaic Turner ie 46XX/45XO or 46XY/45XO
(here gonadectomy is a must for prophylaxis against gonadal
malignancy), other forms include the presence of X(i) ie
isochromosome X,
or Xp- ie deletion of short arm of chromosome X or
X(r) ie ring chromosome X
Other investigations include growth hormone, thyroid
hormones, skeletal survey and bone age, ****bolic screen,
nutritional assessment and other organ function analysis
routine CBC, LFT and RFT etc










Case 17

A 15-Year old boy was reviewed at a school for moderate learing


difficulties
His Height was more than 97th centile
his weight was on the 50th centile
He had bilateral testicular volumes of 2 ml
with no pubic or axillary hair
He had a mild scoliosis
but no other skeletal abnormalities
His mid parental predicted height centile was the 50th
???What is the most likely diagnosis and what is the differential
diagnosis

Most likely diagnosis is Klinefelter syndrome (chromosome
pattern is 47,XXY or 48, XXXY, or 49, XXXXY etc)
Differential diagnosis includes Constitutional familial tall stature
(excluded by 50th percentile mid-parental height), Kallman
syndrome (eunchoidism ie span more than height associated
with anosmia), Marfan syndrome (no mental retardation with
dislocated eye lens or high myope with aortic artery affection,
regurge or dilatation or aneurysm)














Case 18

Case 18
BB, a 12-year-old boy,
presents with
macroscopic haematuria and oedema 2 weeks
after treatment for impetigo.
His blood pressure is 160/110 mmhg.
Serum creatinine is 0.18 mmol/L,
potassium 6.3 mmol/L.
What is the most probable diagnosis
Comment on
Complement,
microscopic hematuria,
penicillin treatment,
antihypertensive therapy,
diet


This boy has a typical history of post-streptococcal glomerulonephritis.
The complement is low in 90% and returns to normal within 8-12
weeks.
It is quite common to see
persistent microscopic haematuria after 6 months,
and this does not indicate a bad prognosis.
However, the persistence of significant proteinuria (> 0.6 gm)
would be a bad prognostic factor.
Treatment with penicillin will decrease the
infectivity for contacts,
but will not alter the prognosis.
Salt and water accumulation is a major cause of the hypertension,
and the addition of a diuretic
with subsequent loss of oedema
often assists in control of the hypertension.
The renin level is usually normal.
This boy does require dietary restriction with no added salt,
low potassium, and protein restriction (0.8-1.0 g/kg).




Case 19
TT, a baby aged 3 weeks,
has prolonged twitching episodes
and is found to have
a serum Ca level of 1.2 mmoi/L.
This is not corrected by
appropriate intravenous calcium infusion.

What would you do?
(A) Double the rate of calcium infusion
(B) Give a large dose of vitamin D orally
(C) Check plasma magnesium levels and give an
intravenous magnesium preparation, while
continuing calcium infusion
(D) Make sure that the correct dose was given
(E) Check plasma phosphate levels
Clearly one should make sure that the correct dose was
given and men check plasma magnesium and
phosphorus, as hypomagnesaemia or
hyperphosphataemia could be the reason for lack
of response. and Increasing calcium or
vitamin D would not be appropriate until you have
looked into these other matters











Case 20

Baby JJ aged 3 months presented with cough,
dyspnoea, and low grade fever for 2 days.
On examination the respiratory rate was 40 breaths per
minute and the alae nasae were flared. The chest was
resonant to percussion, the breath sounds were diminished
and fine crackles were heard over both sides of the chest.
These signs indicate:
(A) Pneumonia
(B) Pleural effusion
(C) Lower airways obstruction
(D) Pulmonary collapse


The correct answer is (C). Fine crackles heard
diffusely over a chest which is resonant to
percussion indicates lower airways obstruction and
in this age group a diagnosis of acute viral
bronchiolitis is most likely. (A) If pneumonia
involved both lungs the infant would be
desperately ill, the chest would not be resonant to
percussion and areas of bronchial breathing would
probably be heard. (B) In pleural effusion, breath sounds are
usually absent or very soft and the percussion note stony dull.
(D) Pulmonary collapse is localized as are the signsreduced
air entry and fine crackles. If the area of collapse is large, the
heart and mediastinum is displaced to the ipsilateral side
















Case21

Abdul, a baby of middle Eastern origin,
aged 6 months
presents
with an afebrile convulsion.
Investigations show
a serum calcium of 1.15 mmol/L,
phosphorus of 1.2 mmot/L
and alkaline phosphatase of 500 IU.
Which is the most likely diagnosis ...
Familial hypophosphataemic rickets
Idiopathic hypoparathyroidism
Nutritional rickets
Cystinosis
Chronic renal failure
The correct answer is (C) Nutritional rickets is
still quite common in such infants, even when
breast fed, as mothers may also be vitamin D
deficient. (A) Familial hypophosphataemic rickets
is unlikely to be associated with hypocalcaerm'a.
(B) Hypoparathyroidism is associated with a high,
rather than low, phosphate level and alkaline
phosphatase is not usually raised. (E) A patient
with chronic renal failure also would have a high
serum phosphate level. (D) Cystinosis is a much
rarer condition, but should be considered if (C)
can be excluded












Case 22

Effie, a 13-month-old girl, presents with a 2 day
history of increasing pallor, lethargy and irritability.
She has been febrile and jaundiced for 24 hours and
her urine is noted to be very dark. She had neonatal
jaundice for which no treatment was required. She
has a temperature of 38.5C and the spleen is
palpable 2 cm below the lower costal margin. The
Hb is 60 g/L with 20% reticulocytes and the white
cell count is 14.0 x 109/Lwith 10% neutrophils and
bands. Which of the following statements are
correct?
(A) The fact that she is jaundiced with dark urine
indicates biliary obstruction
(B) A blood culture should be performed
(C) There may be difficulty in cross-matching
blood for transfusion
(D) The G-6-PD screen is likely to be positive

The correct anwers are (B) and (C).
The story of increased pallor associated
with jaundice and dark
urine suggests acute haemolysis with
haemoglobinuria.
(A) Acute obstructive jaundice
would be uncommon at this age and not
associated with gross anemia.
(B) The fever and high white cell count
may reflect infection which
could be a precipitating cause of haemolysis.
Fever may however be secondary
to intravascular haemolysis
and the elevated white count
reflect increased marrow activity in response to haemolysis.
(C) The Coombs test is likely to be
positive in which case the antibody
is likely to cross react and make a 'compatible' cross match
unlikely.
(D) G-6-PD deficiency is X-linked
and a girl is unlikely to present with acute haemolysis
Case 23
A baby is born by spontaneous, vaginal delivery at 35 weeks'
gestation
following rupture of membranes 30 hours before birth.
The mother was well and not treated with antibiotics during
labour.
A vaginal swab was taken on admission.
The baby was well and
the mother wishes to be discharged home
as soon as possible.
The MOST appropriate course of action
for the baby at this time is which of the following?
Discharge home or Observe in hospital for 48 hours or
Await result of vaginal swab and treat the baby according to this
or Perform an infection screen and treat with intravenous
antibiotics if
results suggestive of sepsis
or Perform an infection screen and treat with intravenous
antibiotics for
at least 48 hours pending results


E: Perform an infection screen
and treat with intravenous antibiotics
for at least 48 hours pending results
Risk factors for neonatal sepsis include
rupture of membranes greater than 12-24 hours,
intrapartum maternal pyrexia (> 38C),
fetal tachycardia,
chorioamnionitis,
pre-term birth
and maternal colonisation with group B Streptococcus.
In this case there are two risk factors:
prolonged rupture of membranes,
and pre-term birth.
Each risk factor represents
approximately 1-2% risk of infection to the baby.
Ideally the mother should have received intra-partum antibiotics
and, if given greater than 4 hours prior to birth,
the baby could have been considered adequately
treated in the absence of other risk factors.
In the absence of this, most guidelines
suggest investigating well babies
for sepsis and treating
if there are two or more risk factors.

Case 24
A well, breast-feeding term infant
presented with jaundice at 36 hours of age.
The serum bilirubin was 286 moI/L,
direct Coombs test negative,
blood film showed spherocytes and reticulocytes,
the baby's blood group was A rhesus negative
and mother's blood group O Rhesus negative.
Which of the following is the MOST likely diagnosis?
A Rhesus haemolytic disease
B ABO incompatibility
C Hereditary spherocytosis
D Physiological jaundice
E Breast milk-associated jaundice

The correct answer is
B: ABO incompatibility
The bilirubin level at this age is too high
for physiological or breast milk-associated jaundice,
which both tend to present later.
Rhesus haemolytic disease is not possible
as the baby is rhesus negative.
Spherocytes and reticulocytes
are seen commonly
on the blood film in ABO incompatibility.
The direct Coombs test is
sometimes negative in ABO incompatibility,
reflecting a low concentration of antibody on red blood cells










Case 25
A girl infant has profound hypotonia and
required intubation and positive pressure ventilation
immediately after birth.
She was born at term following a pregnancy
complicated by polyhydramnios and reduced fetal movements.
The chest X-ray shows small,
but clear, lung fields.
What is the MOST likely diagnosis?
A trisomy2I
B Neonatal encephalopathy
C Congenital myotonic dystrophy
D Cervical spinal injury
E Spinal muscular atrophy


C: Congenital myotonic dystrophy
All options may cause neonatal hypotonia.
Reduced fetal movements and polyhydramnios
(due to reduced fetal swallowing)
suggest a severe neuromuscular disorder.
The small lung fields and ventilatory requirement
suggest pulmonary hypoplasia,
which may be secondary to severe neuromuscular disorders,
which have an early in-utero onset
(due to reduced fetal breathing movements).
This makes neonatal encephalopathy
and cervical spine injury unlikely
because both of these nearly
always occur secondary to an insult
close to the time of birth.
Although cases of spinal muscular atrophy
with fetal onset have been described,
they are extremely rare.
Congenital myotonic dystrophy is therefore the best answer.







Case 26

CASE 26
A term baby born after spontaneous vaginal delivery is noted to be pale.
Full blood count shows hemoglobin 5.4 g/dL;
white cell count 15.1 x 109/L;
platelets 286 x 109/L.
The blood film is normal.
The baby remains well and is transfused
with packed red blood cells.
A cranial ultrasound scan is normal.
In order to
establish the cause of the anaemia,
the NEXT investigation
should be which one of the following?
A Abdominal ultrasound scan
B Coagulation screen
C Apt's test
D Kleihauer test
E Bone marrow biopsy

D: Kleihauer test
The most likely diagnosis is
fetomaternal bleed leading to anaemia.
This can be acute or chronic.
The Kleihauer (Kleihauer-Betke) test
is on maternal blood
and detects the presence of fetal red blood cells,
thus confirming or refuting the diagnosis of a fetomaternal bleed.

The Apt's test is used if there is gastrointestinal bleeding
to distinguish between maternal and baby's blood
as the cause of this.










Case 27

Case 27
A well 3-week-old baby born at term
is referred to hospital with a discharging umbilicus.
The cord separated at 10 days and
there is no peri-umbilical swelling or erythema.
There is a small red mass at the site of cord separation,
which is discharging a small amount of yellow fluid.
The GP had taken a swab of this which
grew Staphylococcus epidermidis.
The MOST appropriate course of action
is which of the following?
A Reassure parents and review in 1-2 weeks
B "freat with oral antibiotics
C Treat with intravenous antibiotics
D Arrange an abdominal ultrasound scan
E Refer to a paediatric surgeon

A: Reassure parents and review in 1-2 weeks
The most likely diagnosis is umbilical granuloma. This is much more
common than other possibilities, such as patent urachus and patent
omphalomesenteric duct, which may present with discharge of urine or
meconium. Umbilical granulomas are usually red and may discharge
serous fluid or pus. Treatment with antibiotics is only necessary if
omphalitis is a possibility, with peri-umbilical erythema in an unwell
baby. Normal skin flora, such as Staphylococcus epidermidis, do not
require antibiotic treatment. Cauterisation with silver nitrate may be
considered but most umbilical granulomata are self-resolving within a
few weeks.








Case 28

Case 28
A woman develops chickenpox 2 days before giving birth to a
healthy term baby. Which of the following is the treatment of
choice?
A No treatment necessary for mother or baby
B Treat mother with aciclovir
C Treat mother with varicella zoster immune globulin only
D Treat baby with varicella zoster immune globulin only
E Treat baby with varicella zoster immune globulin and aciclovir
if chickenpox develops


immune globulin and zoster - Treat baby with varicella : E
aciclovir if chickenpox develops
Perinatal chickenpox is a serious and potentially life-threatening
disease.
Approximately 25% of babies will become infected
if their mother develops chickenpox in the peri-partum period.
Maternal chickenpox within 5 days of birth presents the greatest risk,
as there is insufficient time for transplacetta! transmission of antibodies.

Administration of varicella zoster immune globulin (VZIG)
within 72 hours of exposure has been shown to prevent anil attenuate
chickenpox. Development of symptoms in the baby may be as ear:
as 3 days after the onset of the maternal rash, but may be up to 16 days.
Mortality of untreated cases of perinatal chickenpox
has been reported to be as high as 30%,
therefore treatment with aciclovir is recommended if signs of this
develop.

















Case 29

Case 29
A 7-day-oId term boy is admitted to hospital
with bruising and
bleeding from the umbilical stump.
He had been born at home
and was not given vitamin K after birth.
The full blood count is
normal,
but prothrombin time and activated partial
thromboplastin time are elevated.
What is the BEST treatment
for the baby?
A No treatment
B Intramuscular vitamin Koniy
C Intravenous vitamin K only
D Intravenous vitamin K plus fresh frozen plasma
E Intramuscular vitamin K plus fresh frozen plasma


D: Intravenous vitamin K plus fresh frozen plasma
The diagnosis is haemorrhagic disease of the newborn (vitamin K-
deficiency bleeding).
This usually presents with gastrointestinal bleeding,
bleeding from the umbilical stump and bruising after 2-7 days.
Breast-fed babies not receiving vitamin K prophylaxis after birth are
most at risk.
The prothrombin time is elevated, as is the activated partial
thromboplastin time to a lesser extent.
With active bleeding it is important to give intravenous vitamin K
because this will correct the deficiency quicker than the intramuscular
route.
Fresh frozen plasma should also be given because
even intravenous vitamin K does not correct the clotting times quickly
enough










Case 30

Case Case Case Case 30 30 30 30
An overweight An overweight An overweight An overweight 14 14 14 14- -- -year year year year- -- -old girl old girl old girl old girl
has complained of headache has complained of headache has complained of headache has complained of headache
on awakening for several weeks on awakening for several weeks on awakening for several weeks on awakening for several weeks. . . .
She has papilioedema She has papilioedema She has papilioedema She has papilioedema. . . .
Her blood pressure is normal Her blood pressure is normal Her blood pressure is normal Her blood pressure is normal. . . .
Brain MRI is normal Brain MRI is normal Brain MRI is normal Brain MRI is normal. . . .
Which investigationts Which investigationts Which investigationts Which investigationts
MOST likely to confirm the diagnosis MOST likely to confirm the diagnosis MOST likely to confirm the diagnosis MOST likely to confirm the diagnosis? ?? ?
A Sleep EEG A Sleep EEG A Sleep EEG A Sleep EEG
B CT brain with contrast B CT brain with contrast B CT brain with contrast B CT brain with contrast
C Magnetic resonance spectroscopy C Magnetic resonance spectroscopy C Magnetic resonance spectroscopy C Magnetic resonance spectroscopy
D Lumbar puncture with manometry D Lumbar puncture with manometry D Lumbar puncture with manometry D Lumbar puncture with manometry
E Visual evoked responses and electroretinogram E Visual evoked responses and electroretinogram E Visual evoked responses and electroretinogram E Visual evoked responses and electroretinogram


D: Lumbar puncture with manometry
Benign intracranial hypertension is a headache syndrome
showing raised cerebrospinal fluid pressure in absence of
mass lesion or dilated ventricles, There are usually normal findings on
examination except for papilloedema and occasional Vlth nerve paisy.
The condition requires close monitoring 1 visual acuity as
optic nerve damage can occur as a result of chronically rais pressure.
Treatment can involve repeat lumbar punctures
to remove cerebrospinal fluid,
acetazolamide and analgesics
















Case 31

Case 31
A I4-month-old girl with a history of eczema develops
generalised urticaria, wheeze and severe dyspnoea shortly
after
eating some peanut butter for the first time. What is the
MOST
appropriate initial treatment?
A Adrenaline intramuscularly
B Adrenaline intravenously
C Hydrocortisone intravenously
D Chlorpheniramine intravenously
E Chlorpheniramine orally

A: Adrenaline intramuscularly
The history clearly describes an episode of anaphylaxis.
Adrenaline is the most important single drug in anaphylaxis. It
reverses upper and lower airway oedema, causes
bronchodilation, increases blood pressure and causes
peripheral vasoconstriction, reducing capillary leak. It should be
given by the intramuscular route in most cases. The intravenous
route should only be used by those experienced in this method
and at a concentration of no greater than 1 in 10,000, with ECG
monitoring. The prevalence of peanut allergy in childhood has
tripled over the past decade and it now affects 1.5%. Up to 80%
of children with peanut allergy react on the first apparent
ingestion, suggesting prior occult sensitisation. The median age
at onset is 2 years and only 20% of young children with mild
allergy can expect to grow out of it. A management plan for nut-
allergic children which includes avoidance advice and a tailored
patient-heid self-treatment plan, together with regular follow up,
can reduce the number of further nut-induced reactions. Future
potential treatments include use of anti-lgE and desensitisation
with modified peanut allergen.



Case 32

Case 32
An 8-year-old boy is found at night
making salivatory and gurgling noises.
He is unable to speak
but seems conscious.
His face is twitching on the left side.
Which investigation is MOST likely to confirm the diagnosis?
A Brain CT with contrast
B Brain MRI
C EEG
D Thrombophilia screen
E Lumbar puncture


C: Electroencephalogram (EEG)
This is the typical presentation of a child with rolandic epilepsy,
also termed benign childhood epilepsy with centrotemporal
spikes. It is also common to have brief daytime hemi-facial
twitching with increased salivation. This is a common condition
accounting for about 15% of children with afebrile seizures. It
can be recognised on clinical grounds and has a distinctive EEG














Case 33

Case 33
A newborn presents with a weak suck and weak cry.
There is a fluctuating ptosis and hypotonia.
The baby has attacks of apnoea.
The mother has no medical history of note.
The tests for anti-acetylcholine receptor antibodies are negative
and there is a decremental response
on repetitive nerve stimulation electromyography.
The MOST likely diagnosis is?
A Transient neonatal myasthenia
B Autoimmune myasthenia
C Congenital myasthenia
D Mobius syndrome
E Spinal muscular atrophy type 1

C: Congenital myasthenia
The clues to the diagnosis of myasthenia are the fluctuation of
weakness and the decremental response with repetitive nerve
stimulation EMG.
There are three main groups of myasthenic syndromes:
the autoimmune form (often known as juvenile myasthenia),
congenital myasthenia gravis (sometimes called genetic), a
nd transient neonatal myasthenia.
The latter is due to transplacental passage of antibody to the
acetylcholine receptor in mothers with myasthenia gravis.
Clearance of antibody can take 5 months after birth. Congenital
myasthenic syndromes are a heterogeneous group and the defect can
be pre-synaptic, synaptic and post-synaptic.
It is important to remember these in the differential diagnosis of apnoea
in the newborn. Spinal muscular atrophy type 1 involves degeneration of
motor neurons.
There is symmetrical weakness caused by denervation, with associated
muscle atrophy.
The neonate often will present with hypotonia,
but the face is usually strong1 full movement.
There is no loss of sensation.
The EMG *****strates a neuropathic reduced recruitment of voluntary
muscle unit potentials.
Molecular analysis is now used to make the diagnosis
and over 95% of cases have deletions in the survival motor neuron
(SMN) gene.
Mobius syndrome characterized by biiateral facial weakness.
There is often paralysis of the abducens nerve associated with it.


Case 34

Case 34
The following signs are seen in a child with an ischaemic stroke:
contralateral hemiparesis
with face and arm more affected than the legs;
horizontal gaze palsy and hemisensory deficits;
language and cognitive deficits including aphasia and apraxia;
homonymous hemianopia.
---------------------------------------------------------------
Which artery is the MOST likely to be involved?
A Common carotid
B Middle cerebral
C Anterior cerebral
D Posterior cerebral
E Posterior cerebellar
B: Middle cerebral
Common carotid occlusion may be asymptomatic.
The anterior cerebral arta occlusions present with contralateral
hemiparesis
involving the leg to a greater extent than the arm and face.
There can be a contralateral grasp relit and gait disorders.
Urinary incontinence is a feature.
Posterior cerebral arts occlusions involve contralateral homonymous
hemianopia.
There maybe memory loss and dyslexia.













Case 35

Case 35
-------------------
AHA a four-year-old boy,
presents with oedema.
His 24 hour urine protein is 3 gm,
serum creatinine is normal,
and serum albumin is low at 21 g/L.
There is no haematuria.
He does not respond to prednisolone 2 mg/kg/day for 4 weeks.
----------------------------------------------------------------------
Which of the following statements is correct?
(A) A renal biopsy is indicated
(B) He has Alport's syndrome
(C) Focal segmental glomerulosclerosis is likely
(D) He has a 80% chance of remission with
cyclophosphamide
(E) Recurrence of nephrotic syndrome with a
kidney transplant is < 5%

The correct answers are (A) and (C). As he is steroid resistant a renal
biopsy is indicated (A), and his most likely diagnosis is focal segmental
glomerulosclerosis (C). Alport's syndrome presents with microscopic
haematuria and proteinuria, and nephrotic syndrome does not occur (B).
The response of focal segmental glomerulosclerosis to
immunosuppression is low (D), and the risk of recurrence in a
subsequent kidney transplant is approximately 30% (E).












Case 36

Case 36
========================
A child presents with a facial palsy several weeks
after a flu-like illness she had in the school holidays.
She also has musculoskeletal pains and headache on direct
questioning.
On examination there is also splenomegaly and generalized
lymphadenopathy.
=========================
Which of the following investigations will be
the MOST useful to aid diagnosis?
A Borrelia antibody seroiogy
B Magnetic resonance imaging scan
C Anti-basal ganglia antibodies
D Stool culture for viruses
E Serum treponemal serologic tests

A: Borrelia antibody serology
This clinical presentation is consistent with the diagnosis of Lyme
disease. This is a multisystem disorder caused by a tick-transmitted
spirochete, Borrelia burgdorferi. Localized infection occurs following a
tick bite, which is often forgotten about or not noticed. This is followed
by disseminated infection for up to 9 months. Late or persistent
infection can occur for months or years. Neurological manifestations
include progressive encephalomyelitis focal encephalitis, dementia,
seizures and polyneuropathy.











Case 37

Case 37
============================
A 3-year-old boy presents with frequent falling.
There is
evidence of delay in motor milestones.
He walked at 22 months.
On examination he has a shuffling gait and foot drop.
He has difficulty climbing stairs.
Serum enzymes are normal.
Nerve conduction studies are normal.
CSF exam is normal.
EMG
reveals denervation and paucity of movement.
=====================================
The MOST likely diagnosis is which of the following?
A Type 2 spinal muscular atrophy
B Parkinson's disease
C Werdnig-Hoffman disease -
D Emery-Dreifuss muscular dystrophy
E Kugelberg-Welander disease

E: Kugelberg-We lander disease
The usual onset of Kugelberg-We lander disease is after 2 years and
children have often walked late. There is stow deterioration that results
in scoliosis and wheelchair dependence. Type 2 spinal muscular
atrophy, Werdnig-Hoffman disease, and Kugelberg-Welander disease
are all forms of spinal muscular atrophy, each having degeneration in
the anterior horn cell the spinal cord, and in some cases the motor
nuclei a'the brainstem.Typeli the most severe form and can present with
reduced fetal movements or neonatal hypotonia. Type 2 is intermediate
seventy but children are unable to walk unaided. Muscle biopsy and
molecular genetic studies the SMN gene confirms the diagnosis.
Parkinson's disease rarely presents during childhood






Case 38

Case 38
=========================
An infant presents at age 4 months with hypotonia,
weakness and constipation.
On examination the pupillary responses are poorly reactive.
Investigation reveals incremental response
on rapid (20-50 Hz) repetitive nerve stimulation
and abundant small motor unit potentials on electromyography.
Tensilon test is negative.
What is the MOST likely diagnosis?
A Spinal muscular atrophy
B Congenital myotonic dystrophy
C infantile botulism
D Congenital myopathy
E Myasthenia gravis

C: Infantile botulism
The conditions listed above can all present with peripheral hypotonia.
Infant tulism most commonly presents at 2-6 months with the symptoms
escribed above. Oosthdium botuiinum is a Gram-positive spore-forming
;anism found in soil, agricultural products, and honey. Respiratory
failure n occur. The toxin is released and binds irreversibly to pre-
synaptic olinergic nerve terminals and disrupts the exocytosis of
acetylcholine. sitive stool culture or isolation of toxin is difficult because
of constipation, gement is supportive. Pupillary responses should not
be affected in the lerconditions. A positive Tensilon test would be
expected with myasthenia ndromes. The EMG in congenital myotonic
dystrophy may *****strate yotonic potentials firing at high rates that wax
and wane in frequency and e









Case 39

Case 39
=======================
An 8-year-old boy presents with an 8-week history of swelling of both
knees,
and a dry rash over his elbows and knees.
His father has dry skin patches in his scalp.
He also is found to have swelling of two of his toes.
His GP has been treating him for a fungal infection of his nails.
===================================
What is the MOST likely diagnosis?
A Enthesitis-related juvenile idiopathic arthritis
B Psoriatic arthritis
C Oligoarticular juvenile idiopathic arthritis
D Juvenile dermatomyositis
E Inflammatory bowel disease related arthritis

B: Psoriatic arthritis
Psoriatic arthritis can present with few or several joints affected. It can
occur without the classical psoriatic rash but occasionally a psoriatic-I
ike rash as described here can be seen. Dactylitis is a feature of
psoriatic arthritis and is rarely seen in other types of juvenile idiopathic
arthritis. The father's dry scalp patches may possibly be psoriatic and
family history is often seen. The nail problem may be psoriatic and
typically pitting is seen. In enthesitis-related arthritis, enthesitis and
tendonitis are features - which are not seen in this patient. Enthesitis is
inflammation at the site of tendon insertion into bones such as the
posterior heel.'












Case 40

Case 40
====================
A 10-year-old boy who was previously well
presents with a 2-week history of malaise and headache,
with pleuritic chest pain, cough and fever over the past 3 days.
Five days of treatment with oral penicillin has made no
improvement.
On examination there is a small area of stony dullness at the
right lung base.
======================================
The MOST likely diagnosis is which of the following?
A Lymphoma
B Mycoplasma pneumoniae pneumonia
C Pneumococcal pneumonia
D Pneumocystis carinii pneumonia
E Staphylococcal pneumonia


B: Mycoplasma pneumoniae pneumonia
Mycoplasma pneumoniae is a common cause of atypical pneumonia
among children. The clues in the history are malaise and headache,
more mmon in Mycoplasma infection, together with the age of the child
and lack responsiveness to penicillin (Mycoplasma lack a cell wall).
Macrolide antibiotics are the treatment of choice. Pneumocystis carinii
infection usually affects children with a severe underlying immune
deficiency, such as severe combined immunodeficiency syndrome
(SCID), di George syndrome, and post bone-marrow transplantation
(BMT).


















Case 41

A 10-year-old girl presents with headache. It is unilateral,
pulsating and rates6/10 on the pain scale. The headache is
worse on physical exercise and helped by sleep. Sometimes she
sees photopsias. There is often nausea. What is the MOST likely
diagnosis?
A Cluster headache
B Tension headache
C Hydrocephalus
D Chronic paroxysmal hemicrania
E Classical migraine

E: Classical migraine
headache disorders are common in childhood. This is a case of
classic migraine with aura of flashing lights. Cluster headaches
are more common in adults, most often occurring in those in
their late 20s and rarely in children, triocficity is the predominant
feature with chronic episodes of headaches
for 2-3 months; there is usually no aura, and the pain is
excruciating ten in the trigeminal distribution and may be
associated with lacrimation, sweating, ptosis and nasal
congestion. Tension headaches are diffuse and often described
as pressing or a being like a tight band; auras are rare. Chronic
paroxysmal hemicrania is similar to cluster headache but the
pain is usually in or around the orbit; headaches last for a short
time (5 minutes) but occur many times a day. Hydrocephalus
can present as headache. This is felly worse in the morning.
There may be associated gait abnormalities and cranial nerve
signs notably VIth nerve palsy and diplopia. There is often loss
of fine motor coordination. Papilloedema is frequently present







Case 42

Case 42
A 3-month-old baby has a history of wheeze and coughing
usually after feeds, since birth. Her weight is normal and on
examination she has a Harrison sulcus and a hyper-expanded
chest. Which of the following investigations would be the MOST
useful?
A Flexible bronchoscopy
B Computed tomography (CT) scan of the chest
C Ciliary brushing for motility
D Sweat electrolytes
E Upper gastrointestinal (GI) contrast study

E: Upper gastrointestinal (GI) contrast study
This child presents with a picture of lower airway obstruction
that is associated with feeds, implying the presence of gastro-
oesophageal reflux disease, an H-type tracheo-oesophageal
fistula (TOF) or viral-induced wheeze. The most useful
information would come from an upper GI contrast study which
should include a tube oesophogram to exclude a TOF. Flexible
bronchoscopy is poor for excluding TOF, and ciliary brushing is
used only for the diagnosis of primary ciliary dyskinesia. It
would be sensible to screen for cystic fibrosis .






















43 Case
A 2-year-old boy has a history of lethargy and falling asleep
during the day. His mother reports that he snores loudly. Which
is the MOST useful investigation?
A Arterial blood gas
B Electrocardiogram (ECG)
C Micro laryngobronchoscopy (MLB)
D Overnight oxygen saturation recording
E Lateral upper airways X-ray


D: Overnight oxygen saturation recording
This child has a history suggestive of sleep apnoea. The
presentation can be subtle and families will often not report
pauses in breathing whilst asleep unless specifically
questioned. Other features which are suggestive are early
morning headaches (due to high arterial CO2 partial pressure
(Pco2) and an 'adenoidal' voice. It is most often caused by
adenotonsillar hypertrophy but neuromuscular conditions must
also be considered. Arterial blood gas may show high Pco2 and
upper airway XR or MLB may reveal large adenoids, but most
information will be obtained from overnight oxygen saturation
monitoring. This will detect periods of desaturation and apnoea,
and linked to analysis of chest and abdominal movements can
help distinguish between obstructive causes (tonsils and central
causes (eg brain stem tumour


















Case 44

A 3-year-old girl presents with a 3-week history of fever,
with daily spikes of 40C, a pink rash that varies, painful
muscles, some swelling of her wrists for 7 weeks, and
generalized lymphadenopathy. Investigations reveal anaemia,
with a high white cell count, raised platelets, very high
erythrocyte sedimentation rate and C reactive protein.
Antinuclear antibody, double-stranded DNA, and rheumatoid
factor are negative. Which of the following is the MOST
likely diagnosis?
A Systemic lupus erythematosus
B Acute lymphoblastic leukaemia
C Systemic onset juvenile idiopathic arthritis
D Parvovirus infection
E Neuroblastoma

C: Systemic onset juvenile idiopathic arthritis
These are all the classical findings in systemic onset juvenile
idiopathic arthritis. More than a 2-week history of spiking
fevers is one of the criteria needed for the agnosis. The rash
is salmon pink and can vary in intensity over the day, being
prominent at times of fever. Generalised lymphadenopathy is
a common feature. Arthritis has to be present for more than
6weeks in systemic onset juvenile idiopathic arthritis.
Systemic lupus erythematosus usually has a positive
antinuclear antibody and normal C reactive protein. Platelets
are usually normal or low in acute lymphoblastic leukaemia.
The history is too prolonged for parvovirus, and rash is
unusual in neuroblastoma, as is high platelet count.






Case 45


Case 45

A new born infant has ambiguous genitalia.
Would you?
(A) Arrange urgent karyotype
(B) Take blood for serum electrolytes
(C) Measure blood glucose
(D) Obtain a surgical opinion as soon as possible
(E) Take blood for plasma cortisol

The correct answers are (A), (B) and (C). CAH
has to be excluded as a matter of urgency. If not
recognized and treated with both glucocorticoids
and mineralocorticoids this infant will die of salt
wasting. Early signs of salt wasting are a low serum
sodium and elevated serum potassium, together
with a low blood glucose. A karyotype 46XX
makes the diagnosis likely. (D) A surgical opinion
is not required until the diagnosis of CAH has
been excluded and the infant has been stabilized
on treatment. The diagnosis of CAH is made on
the basis of an elevated plasma 17 hydroxy progesterone.
(E) A plasma cortisol estimation, although low in CAH is not helpful in
diagnosis.










Case 46

Bianca, a 5-year-old girl has developed left-sided
red swollen eyelids after cold-like symptoms for a
week. There is local pain and she is feverish and
unwell. Parting the lids reveals that the eye is
protuberant and its mobility seems limited. Her
vision seems alright. Would you?

a)Advise a cold compress, bed rest and a 10day course of oral
antibiotics

b)Arrange a hospital admission and intravenous antibiotics

c)Organize urgent hospital admission, opthalmology and ENT opinions
and a CT scan of the sinuses and cranial-*******s

d)Tell the parents that the vision in that eye at great risk unless the
abscess is immediately drained and send them to the local emergency
ward with a letter to that effect

The correct answer is (C). This is orbital cellulitis,
probably at the subperiosteal phase
(A) and (B)
denotes that Vigorous and prompt medical and probably
surgical therapy is required, but only after careful
clinical and radiological assessment as stated in (C). as regards
(D) The
outcome is usually good, so unnecessary anxiety
can be avoided.









Case 47


Case 47

Case 47
OO, a 9-month-old infant, presents with a2 month history of
increasing frequency of bowel actions up to 7 times daily. The
motions were described as loose to runny paste, and
occasionally explosive. He was totally breast fed until 6
months of age when cereal, fruit and yoghurt were
introduced. At 7 months he was weaned to a cow's milk
containing infant formula. General examination was a normal
with his weight on the 50th percentile and height on the 75th
percentile. What is appropraite to do?Would you? Start him
on a cow's milk protein free diet or Collect stool specimens
for microscopy and culture or Enquire about fruit juice intake
or Do a breath hydrogen test for sucrose malabsorption

The correct answers are to Collect stool specimens for
microscopy and culture and Enquire about fruit juice intake
and also Do a breath hydrogen test for sucrose
malabsorption. This type of presentation is common and it is
important to investigate the possible causes before embarking
on dietary change. Possibilities include a parasitic infection
(giardiasis, cryptosporidium), sucrase deficiency or a post
infectious diarrhoea. It is also appropriate to investigate the
type of diet the infant is having as certain foods or fluids in
excess such as apple juice or sorbitol containing drinks can
cause diarrhoea Sucrase deficiency can present at this age
with no abnormal clinical findings. It can occur as a primary,
congenital abnormality or secondary to infectious enteritis. On
the other hand, Cow's milk protein intolerance is uncommon
and it is important to first exclude the more common non-
immune causes of diarrhoea

Case 48


Case 48
JJ, a 5-month-old girl, was well until 12 hours ago when she
began vomiting. She refused to feed and at times seemed to be
getting spasms of pain. Examination revealed a pale and
lethargic child with mild abdominal tenderness, but no definite
abdominal mass was palpable. The rectum was empty and
contained no blood. What would you primarily do? Would you?
Perform a lumbar puncture to exclude meningitis or Put her on
gastrolyte to avoid dehydration or Arrange an air enema to
confirm and reduce a suspected intussusception or Obtain a
paediatric surgical opinion because appendicitis in an infant is
difficult to diagnose

The correct answer is to arrange for an air enema as
Intussusception is the most likely diagnosis here. Failure to
palpate an abdominal mass or to pass bloody stools does not
exclude intussusception. An air enema not only will confirm
the diagnosis, but also will reduce it. Meningitis can occur in
any age group and has some of the features described above,
but does not usually produce spasms of (abdominal) pain. If a
diagnostic enema to exclude intussusception was negative, a
septic cause should be considered. The absence of significant
diarrhoea makes gastroenteritis unlikely. Gastrolyte would not
be helpful in this situation. Appendicitis is extremely rare in
this age group, and is difficult to diagnose until the disease is
advanced, that is, peritonitis is present. However,
intussusception is a much more likely explanation for the
symptoms. The paediatric surgeon would suggest an air enema
be performed.








Case 49


Case 49
LL, an apparently healthy 6-week-old baby, has remained
mildly jaundiced since birth. Clinical examination reveals no
additional abnormalities. What would you prefer to do? Would
you? Arrange for liver function blood tests and review the
baby in 2 weeks' time or Refer LL to a local paediatrician
meanwhile requesting liver function blood tests so that results
will be available when she sees her in 3 weeks' time or
Telephone the local paediatrician and ask that the baby be
seen within the next 24-48 hours

The correct answer is to Telephone the Paediatrician for
urgent examination of the baby as Prolonged jaundice in
infancy is always a potential emergency. A paediatrician
should see this baby within 48 hours. The jaundice may well
be due to a benign cause, however metabolic diseases and or
biliary obstruction leading to hepatocellular injury need urgent
exclusion. Because conjugated bilirubin is water soluble and
infants have high fluid intakes, jaundice due to complete
biliary obstruction may be relatively mild. If the baby has
biliary atresia, a delay of a few weeks may lead to
considerable increase in the severity of hepatic cirrhosis, as
well as greatly decreasing the chances of response to surgical
relief of the biliary obstruction













Case 50

Case 50

A 2 year old boy presents with a murmur heard in both
systole and diastole at the upper sternal edge, which
disappears upon lying down. Physical examination was
otherwise normal. He is a well, asymptomatic child and there
are no signs of cardiac failure.You are told that his second
cousin had a small ventricular septal defect, which closed
spontaneously, and that his uncle had a heart attack aged 45.
What do you consider to BEST management plan? Would you?
: Refer for echocardiography and specialist opinion from a
consultant pediatric cardiologist or perform an
electrocardiogram, chest radiograph, and oxygen saturations
and then refer for echocardiography or refer for genetic
counselling and possible gene mapping studies or reassure the
parents that the murmur is innocent or say that you suspect
the murmur is caused by a persistent arterial duct which
should be coil-occluded to avoid the development of heart
failure in future


Actually this is an interesting practical case in terms that
sometimes we get alarmed by simple physiologic normal
phenomena that gives a warning for clinicians to be patient
and not jump into enthusiastic diagnosis, The clue in this case
is that this murmur is more or less localised and also
disappear by change of position "lying flat" and the child is
asymptomatic as well. So the correct answer is that you have
to reassure parents that this is an innocent murmer . This is
typical of a venous hum, an innocent heart murmur, It may be
easy to hear the venous blood flow returning to the heart,
especial1y at the upper sternal edge. This characteristically
occurs in both systole and diastole and disappears when the
child lies flat. Innocent murmurs are the commonest murmurs
heard in children, occurring in up to 50% of normal children.
They are often discovered in in children with a co-existing
infection or with anaemia. Innocent murmurs all relate to a
structurally normal heart and it is clearly important to
reassure the parents that their child's heart is normal. Types
of innocent murmurs include those due to increased flow
across the branch pulmonary artery, Still's murmur, and
venous hums. The murmur should be soft, (with no thrill),
systolic (diastolic murmurs are not innocent) and short, never
pansystolic. The child is always asymptomatic. The murmur
may change with posture as in venous hums




































Case 51

Case 51


A I-month-old baby presents to the ER with breathlessness
and poor feeding. Her mother says she has been getting
worse over the last week. On examination she was found to
have a large liver, a respiratory rate of60/min, and an active
precordium with a soft murmur. Her oxygen saturations are
94% and she has good femoral femoral pulses. What would
you advise as the MOST appropriate treatment? Would you? :
Start digoxin and diuretic and see her in the clinic in 3
months time or Start captopril in casualty and see in the
clinic in 1 month or Refer for immediate cardiac surgery or
Restrict milk feeds and fluids or Start diuretics and admit to
the Ward for observation.

The correct answer is to start diuretics and admit to the
ward for observation as this case is most probably a case of
VSD manifesting with pulmonary congestion and heart failure.
Ventricular septal defects can be small or large, small
defects occur anywhere in the vent septum (peri-membraneous
or muscular) and dont require imminent cardiac surgery since
there is no pulmonary hypertension. About 80-90% of
affected children are asymptomatic. They may have a thrill
at the left lower sternal edge and have a harsh pansystolic
murmur at the lower sternal edge with a quiet P2. Large
defects occur anywhere in the septum and usually associated
with pulmonary hypertension and manifest with heart failure
after the first week of age. There can be either a soft or no
systolic murmur and an apical mid-diastolic ht murmur. They
should be treated medically until about 3 months of age, if
not in severe ht failure. If the defect is still large at that
time, then they should undergo surgical closure. There is
usually no need for cardiac catheterisation or MRI.

Case 52


Case 52

A 1day-old baby who is otherwise asymptomatic presents with
a loud harsh heart murmur at the left sternal edge. There
are no features of heart failure present, the oxygen
saturations are normal, and the electrocardiogram performed
by the resident senior house officer is reported to be normal.
What is the MOST likely diagnosis in this case? Is it : Atrial
septal defect or small muscular ventricular septal defect or
Large muscular ventricular septal defect or Pulmonary stenosis
or Persistent arterial duct

The correct answer is : Pulmonary stenosis; those children
with left to -right shunts have no signs or symptoms on the
first day of life. However, those with outflow obstruction
have a murmur from birlh (Note that VSD usually is not
recognized early after birth). Pulmonary stenosis usually
causes no cyanosis, and all neonates have a dominant right
ventricle, thus revealing no evidence of right ventricular
hypertrophy.


















Case 53

Case 53

PP, an 11-month-old boy who was born at 35 weeks'
gestation, presents with poor weight gain over the preceding
3-4 months, pallor, irritability, and intermittent diarrhoea.
He is breast fed but cereal, fruit and vegetables were
introduced from 6 months of age. He is on the 10th centile
for weight and 50th centile for length. There are no specific
findings other than pallor. Blood count shows a Hb of 70 g/L
with an MCV of 102 fl, and oval macrocytes. The platelet
count is 95 x 109/L and neutrophils show hypersegmentation.
What is the Most appropriate way of thinking to have a clue
for diagnosis Would you? : Do a bone marrow aspirate? You
think that The most likely diagnosis is dietary folate
deficiency Would you have a blood test for His mother Would
you install Treatment until results of serum vitamin B12 and
serum and RBC folate are available.


The correct answers are to do bone marrow aspiration and to
test the mother for same deficiency. This infant has a
macrocytic anemia which may be due to vitamin B12 or folate
deficiency, or a congenital dyserythropoietic anemia. The
latter is rare but will be diagnosed on the BMA performed to
confirm megaloblastosis. Folate deficiency would be unlikely in
a breast fed baby eating vegetables, though his requirements
will have been increased because of his prematurity. The
failure to thrive and diarrhoea could be secondary to
malabsorption from cocliac disease in which case folate
deficiency is possible. At this age a megaloblastic anemia due
to vitamin B12 deficiency would reflect either malabsorption
from congenital PA or low stores at birth with subsequent
inadequate intake in breast milk and diet because of maternal
deficiency. As soon as the diagnosis of megaloblastosis has
been confirmed by BMA, treatment with both vitamin BJ2 and
folate should be commenced. Delaying may result in neurologic
damage if the deficiency is vitamin B12













































Case 54


Case 54

ZZ, a phenotypic female aged 2 years, was found to have a
gonad in the inguinal canal at the time of hernia repair. What
is the most appropriate approach you would consider in that
case? Would you? : Take blood for karotype, Biopsy the
gonad, Arrange a pelvic ultrasound, Measure plasma androgens
and oestrogens and Would you arrange to remove or leave the
the gonads? What gender this baby will be assigned to later
on?

The correct answers are to have a blood for karyotype, and do
gonadal biopsy and pelvic ultrasonography as well and
This is because the ). do gonadectomy ( that gonads remove
most likely diagnosis is androgen insensitivity (testicular
feminization syndrome). The likely karyotype is 46XY and the
gonads testes. Rarely an ovary may present in the inguinal
canal, however pelvic ultrasonography will usually suggest the
lack of a uterus, but a laparotomy may be necessary. The
gonads should be removed as they are likely to become
malignant. The baby should be reared as a female although
being amenorrhic and infertile in future. Hormone studies are
not likely to be helpful in the management. However,
Oestrogen therapy will be required at puberty.








Case 55

Case 55

An I8-month-old girl presents with a 6-month history of weight
loss and cough. Examination reveals a clear chest, cervical
lymphadenopathy and red, tender lesions on her shins. Which
is the MOST useful diagnostic test? Erythrocyte sedimentation
rate (ESR) or Bronchoalveolar lavage or Sputum culture or
Gastric aspirate or Skin biopsy
The correct answer is Gastric aspirate analysis for tubercle
bacilli (stain and culture). The scenario describes a child with
tuberculosis, the red lesions suggesting erythema nodosum.
The presentation can be subtle with few chest signs. The
classic features of fever, lethargy and weight loss can be
absent in children. Acid-fast bacilli are best obtained by
gastric washings, usually on three successive mornings;
bronchoalveolar lavage has a lower yield. The diagnosis can be
confirmed by intradermal tuberculin testing. Initial treatment
consists of isoniazid (for 6 months), rifampicin (6 months) and
pyrazinamide (2 months). Monitor liver function tests and
examine for peripheral neuropathy























Case 56

Case 56

BB an 8-year-old visitor to the area, became drowsy on
Feast Day. For the last 4 weeks he has been jaundiced. His
own doctor has investigated him extensively and concluded he
has infectious hepatitis despite negative serologic tests for
hepatitis A, B and C. What is your opinion in the management
of that case taking into consideration the following options ?
A) to Check his liver function tests today and if his liver
function tests arc similar to those earlier in the week allow
him to stay with his parents over the holiday B) to Admit him
to the local hospital with the intention of performing further
tests at the end of the holiday period C)to Phone the local
paediatrician today and ask him to interrupt his family
activities to see and take over the child care D) to Check his
liver function tests today E) to Check his blood clotting
function today


The correct answers are (C), (D) and (E). Drowsiness in a
jaundiced patient is an ominous
sign. It may signify the development of hepatic
encephalopathy, bacterial sepsis or intracranial
bleeding. (C) It is well worth spoiling your paediatrician
colleague's feast day. (D) and (E) You will aid his assessment
if you can provide him with liver function tests and blood
clotting results, but only if this produces minimal delay in the
consultation. Perhaps surprisingly, a long delay between the
onset of jaundice and development of signs of hepatic
encephalopathy is an indicator of bad prognosis. Even with
second generation tests for hepatitis C infection the cause of
many cases of fulminant liver failure in children remains
unknown.


Case 57


Case 57

A 10-year-old male child with epilepsy is admitted with a
heart murmur. On examination, he is hypertensive, the blood
pressure is greater in the proximal than the distal limbs. He
has a prominent carotid pulse and a left ventricular heave on
palpation. Heart sounds depict an ejection systolic murmur at
the upper left sternal edge and posteriorly over the left
interscapular area. An apical ejection click is audible at the
apex. Palpation of the pulses shows decreased volume in the
femoral arteries. The child has an abnormal chest radiograph
with mild cardiomegaly and a slight abnormality of the inferior
surface of the 3rd, 4th and 5th ribs. His ECG shows mild
left ventricular hypertrophy.
What is the diagnosis?


of all % 4 occurs in Coarctation of the aorta
congenital heart disease. It is more prominent in males than
females and is associated with trisomy 13 and 18, Turner's
syndrome, valproate toxicity and abnormalities of ventricular
septal defect, persistent ductus arteriosus, mitral valve
abnormalities and berry aneurysms of the circle of Willis.
Other associated cardiac abnormalities include bicuspid aortic
valve (70%), mitral valve disease, aortic regurgitation (20%)
and subaortic stenosis. Ninety-eight per cent of coarctations
occur at the level of the pulmonary artery after the
subclavian artery. It is for this reason that, on observation,
the proximal blood pressure varies compared with the distal
blood pressure. The blood pressure in the right arm is often
greater than that in the left arm. Clinically, these children
present with hypertension, prominent carotid pulses,
radiofemoral delay, left ventricular hypertrophy and an
ejection systolic murmur maximum over the posterior left
interscapular area. An apical click over the aortic valve may
be heard. Coarctation of the aorta may be simple (post -
ductal) or complex (pre-ductal or with a septal defect), and
may be associated with aortic stenosis, transposition of the
great arteries or a bicuspid aortic valve.
The ECG and chest radiograph may be normal. However, as
the child enters the first decade, evidence of cardiomegaly,
congestive heart failure, post-stenotic dilatation with a
dilated subclavian artery and rib notching may be noticed.
The ECG may show right ventricular hypertrophy, left
ventricular hypertrophy in infancy and right axis deviation.
Complications of coarctation of the aorta include left
ventricular failure, cerebral haemorrhage, aortic dissection,
renal vascular stenosis and infective endocarditis. This
condition may result in death due to an aortic aneurysm or
rupture in the third or fourth decade of life. It may also
cause premature ischaemic heart disease as a result of
hypertension. If left untreated, 20% of individuals die before
20 years of age and 80% before 50 years of age. Treatment
options include surgical balloon dilatation or the grafting of a
subclavian flap, and should surgical correction not normalize
the blood pressure, further medical management is required.






















Case 58

Case 58

An 8-year-old girl presents with her traveler family
complaining of a sore throat, fever, painful joints, a rash and
abnormal movements. On examination, she looks quite
miserable, with a fever of 38.4C and an erythematous rash
with raised edges. On examination, you note her to have
abnormal rhythmical movements and arthralgia with associated
joint swelling of the left ankle and right elbow. She has
multiple painless subcutaneous nodules under her skin. On
examination, she is tachycardic and hypertensive. On
auscultation, there is an apical mid-diastolic murmur.
Investigations stow a leukocytosis with a raised C-reactive
protein (CRP) level and erythrocyte sedimentation rate (ESR),
and her ECG confirms a tachycardia, a prolonged PR interval
and flat inverted T waves.
What is the diagnosis ???

occurs because of an abnormal Rheumatic fever
immune response to a streptococcal antigenetic component. It
has a latent period of 1-3 weeks and is more common in the
lower socio-economic classes. It peaks at around 5-15 years
of age and affects the blood vessels, joints, nervous system
and subcutaneous tissues. It is characterized as an
autoimmune disease, and there is a risk of rheumatic fever
occurring after infection in 3% of the population. The
recurrence is greater in younger children and increases with
each attack. Rheumatic fever is categorized according to the
Duckett- Jones criteria, which are as follows:
Major: carditis 50%, chorea (Sydenham's) 15%,
polyarthritis (flitting)
70%, erythema marginatum 10%, subcutaneous nodules
(painless) 1 %. Minor: arthralgia, fever, prolonged PR
interval, raised ESR, raised CRP.
Supporting evidence is a previous history of rheumatic fever,
evidence of streptococcal disease from a throat swab, a
raised ASO titre and a positive streptococcal antigen test or
a leukocytosis.
In order to make the diagnosis of rheumatic fever, two major
and/or one major with two minor criteria are required.
Evidence of a recent streptococcal infection with a raised
ASO titre or an anti ribonuclease B level is sufficient.
Exceptions are as follows:
chorea alone is diagnostic
an insidious or late-onset carditis with no other explanation
rheumatic recurrence, one major and one minor criterion
with prior
streptococcal disease that is recurring.
Consequences of pericarditis include heart block, pericardial
effusion, tachycardia, cardiomegaly, pericardial friction rub,
congestive cardiac failure, valvular disease and a Carey-
Coombes apical mid-diastolic rumbling murmur. New heart
murmurs are often audible, including those of mitral
regurgitation and aortic regurgitation. Skin nodules affect the
perivascular tissues and are non-specific lesions resulting from
fibroid degeneration. Medication includes aspirin for the acute
phase, non-steroidal antiinflammatory drugs for the
arthritis, prednisolone for severe carditis, and high-dose
penicillin for immediate management with antibiotic prophylaxis
in the long term. Antibiotics may include penicillin V,
erythromycin or benzyl penicillin. Diazepam and haloperidol
may be required to control the chorea.







Case 59

Case 59

A 6-month-old male infant is admitted with shortness of
breath, oedema, cyanosis and arrhythmia. On examination, he
has oedema with central cyanosis and an irregular pulse. A
displaced apical beat is heard laterally, with a marked right
ventricular heave and bilateral basal crackles in the chest. A
chest radiograph shows gross cardiomegaly. An
echocardiogram confirms mitral valve prolapse, and an ECG
shows a short PR interval with a prolonged widened QRS
complex and an upslurring of this complex. There is right
bundle branch block and a dominant R wave in VI. What is
the most probable diagnosis?

White - Parkinson - Type A Wolff
1 000 / .5 1 has an incidence of ) WPWS ( syndrome
cases and is more common in boys and men. The most common
presentation is with re-entry tachycardia due to an aberrant re-
entry circuit at the bundle of Kent. WPWS displays an atrial
ventricular re-entry tachycardia that may be regular or irregular.
This abnormality shows a disorder of the pathway between the atrial
and ventricular myocardium, and it may present with supraventricular
tachycardia owing to pre-excitation; it therefore presents in regular
arrhythmia as a recurrent supraventricular tachycardia but not a
ventricular tachycardia in 10%. It may be due to an anomalous atrial
ventricular conduction pathway.
WPWS is associated with mitral valve prolapse, commonly seen in
Marfan syndrome, corrected transposition of the great arteries,
Epstein's anomaly (as in this case) and atrial septal defect (ostium
secundum). It is also found in hypertrophic obstructive
cardiomyopathy, but it is not associated with myocarditis, tamponade
or a vein of Galen abnormality. In the adult population, it may be
associated with thyrotoxicosis and lithium ingestion during the
antenatal period. In this case, Epstein's anomaly is associated with
displacement of the septal and posterior leaflets of the tricuspid
valve. This causes atrialisation of part of the right ventricle and
subsequent cyanosis and heart failure.
WPWS is usually diagnosed on the ECG, which shows a short PR
interval, a widened QRS complex and a delta wave. There is a
dominant R in VI with left-sided right bundle branch block pattern in
type A WPWS, whereas in type B there is a left bundle branch block
pattern that is right-sided with no dominant R wave in VI. There may
be associated inverted T waves in the anterior leads. In the older
population, WPWS presents most commonly with atrial ventricular re-
entry tachycardia, which shows regular narrow complex arrhythmia in
70% of cases. However, 10-15% may present in atrial fibrillation and
5% in atrial flutter, and it is rare to have atrial ventricular non-re-
entry circuit tachycardia.
Clinically, infants present in heart failure. The older population may
present with dizziness, dyspnoea and palpitations. Prophylaxis may be
controlled with class 1 C verapamil, and surgical intervention includes
radiofrequency ablation, which has a 90% cure rate. Amiodarone may
be highly effective for all arrhythmias in the refractory period to
prevent development of the accessory pathway but should be avoided
in the blocked atrial ventricular node. Long-term use can lead to
thyroid problems. Flecainide is also successful for dysrhythmias; it is,
however, not used in adults. In infants, it is good maintenance
therapy. Long-term verapamiI treatment may cause hypotension, and
sotalol is avoided in the under-I2-month age group, as is digoxin.













Case 60

Case 60

Twelve year-old boy who has returned from the
Mediterranean after working on a farm presents with a
chronic relapsing fever, sweating episodes and general
arthralgia and anorexia. He has lost a lot of weight and is
currently complaining of constipation. His previous medical
history includes a week's duration of a very high fever with
associated headache, arthralgia, malaise and profuse
sweating. On examination, you find a very thin boy who has a
papular rash throughout, swollen ankles and a left swollen
elbow; the cardiovascular and respiratory examination is
unremarkable. However, on palpation of the abdomen he has
hepatosplenomegaly with inguinal lymphadenopathy and swollen
testes. Investigations show a normal white blood cell count,
but the boy's blood culture has grown a Gram-negative, non-
spore-forming intracellular aerobe. What is the diagnosis?


Brucellosis Brucellosis Brucellosis Brucellosis is a zoonotic disease that is commonly
acquired in the Mediterranean and the Middle East and has a
one to five week incubation period. It is usually contracted by
consuming unpasteurised milk and cheese products or when
working on farms, in abattoirs or in butcher's shops with
goats, sheep, pigs and cattle. The acute febrile s***e shows
a high fever, headache, malaise, profuse sweating and general
arthralgia with a few localizing signs. The chronic phase,
which occurs in 20-60% of cases, presents with relapsing
fever. sweats, arthralgia, malaise, anorexia, weight loss,
constipation and emotional disturbance. Examination often
reveals marked lymphadenopathy with a papular rash.
Abdominal examination may reveal hepatosplenomegaly.
Complications include arthropathy, arthralgia, osteomyelitis,
liver granulomas, granulomatous hepatitis, endocarditis,
meningitis and transverse myelitis. Rare severe complications
include subacute bacterial endocarditis, orchitis, neuritis
(unilateral in 2-10%), neuropathy. cholecystitis and
osteomyelitis of the vertebral sacroiliac joint, spondylitis and
peripheral reactive arthropathy. Medication mcludes
doxycycline or co-trimoxazole, but steroids, streptomycin,
gentamicin and rifampicin have been used. Brucellosis is
caused by a Gram negative, non-spore-forming aerobe
coccobacillus that is small, nonmotile and non-encapsulated.
four species are known in the human, these being, in order of
severity: Brucella melitensis, abortus, suis and canis

S-ar putea să vă placă și